Solución a números primos en una ecuación

Olimpiada Iberoamericana de Matemáticas 2016, problema 1.
Se dirige a una edad de: 16/17

Encuentra todos los números primos p, q, r, y k tales que pq + qr + rp = 12k + 1.

La distribución de los números primos entre los números naturales es muy extraña, sigue resistiendo los avances de los expertos en algunos detalles. Sin embargo, hay patrones que cualquiera puede encontrar de forma sencilla.

En nuestro caso, los números que nos interesan son los productos de dos primos, que, por supuesto, también tienen una extraña distribución dentro del conjunto.

Para empezar, observa que el papel de p, q y r juegan papeles intercambiables, es decir, que en cualquier solución se puede cambiar uno por otro y sigue siendo una solución.

Si suponemos que los tres son impares (y el único primo par es 2), resulta que los tres productos también serán impares, existirán valores n, m, y s de forma que p = 2n + 1, q = 2m + 1 y r = 2s + 1, y la expresión que buscamos será (2n + 1)(2m + 1) + (2m + 1)(2s + 1) + (2s + 1)(2n + 1) = 4nm + 4ms + 4ns + 4n + 4m + 4s + 3 = 4(nm + ms + ns + n + m + s) + 3, mientras que al otro lado de la igualdad será 4(3k) + 1, lo cual es imposible, ya que por uno de los números, al restarle uno, es múltiplo de 4, y el otro no.

Por lo tanto, uno de los tres primos es 2. Supongamos que es r (recordemos que luego podemos intercambiarlos). La expresión queda pq + 2p + 2q = 12k + 1.

De manera similar, tratemos de ver si es 3 uno de los dos. Lo que pasa es que ahora es más complicado, porque hay dos posibles restos al dividir por 3, si no da exacto (1 o 2). Para resumir las expresiones, sólo vamos a fijarnos en los restos. Observa que omitimos toda referencia a la parte que es múltiplo de 3.

Suponiendo que p y q tienen ambos resto 1, pq + 2p + 2q = 1 + 2 + 2 = 5 = 2, mientras que 12k + 1 = 1, por lo que no es posible.

La versión larga sería que p = 3m + 1, q = 3n + 1, y por tanto desarrollaríamos la expresión y todo saldría múltiplo de 3 excepto las operaciones que hemos indicado. También hay que tener en cuenta que 5 = 3 + 2, de forma que es un múltiplo de 3 más un resto de 2 unidades.

Supongamos que uno de ellos tiene resto 1 y el otro 2 (como son intercambiables, da lo mismo uno que otro). pq + 2p + 2q = 2 + 2 + 4 = 8 = 2, mientras que 12k + 1 = 1, de nuevo es imposible.

Por último, si ambos tienen resto 2, tenemos que pq + 2p + 2q = 4 + 4 + 4 = 12 = 0, lo que quiere decir que en ese caso, la primera expresión sería múltiplo de 3, mientras que 12k + 1 = 1, y de nuevo es imposible.

Es decir, que o bien p o bien q debe ser 3, podemos suponer que q es 3 (recuerda que son intercambiables), y ya sólo nos queda conocer p. Nuestra expresión inicial es ahora: 3p + 2p + 6 = 12k + 1, es decir, 5p + 6 = 12k + 1. Vamos a fijarnos en esta ocasión en los restos al dividir por 5. Vemos que en el primer lado de la igualdad es 5p + 5 + 1, por lo que el resto es 1. Pero, para que el otro lado, 12k + 1, tenga el mismo resto, 12k debe ser múltiplo de 5, y, puesto que 12 no es divisible entre 5, k debe serlo. Y como es primo, no queda más remedio que que sea 5.

Lo que deja nuestra igualdad inicial en 5p + 6 = 61, de donde deducimos que p = 11.

Es decir que, p, q y r deben ser 2, 3 y 11 en cualquier orden, mientras que k sólo puede ser 5.

O, de forma extendida, serían 6 las soluciones para (p, q, r, k): (2, 3, 11, 5), (3, 2, 11, 5), (11, 3, 2, 5), (3, 11, 2, 5), (2, 11, 3, 5), (11, 2, 3, 5).

Published by

dimates

Grupo de divulgación matemática de la Universidad de Alicante

Deja un comentario

Tu dirección de correo electrónico no será publicada. Los campos necesarios están marcados *